0 Daumen
3,7k Aufrufe

Aufgabe:

Berechnen Sie den Konvergenzradius der reellen Potenzreihe \( \sum \limits_{k=1}^{\infty} \frac{(-1)^{k}}{4 k+1} x^{k} \) und untersuchen Sie das Konvergenzverhalten an den Randpunkten. Geben Sie den Konvergenzbereich an.


Ich bräuchte da mal ganz Hilfe, da das Thema gar nicht in meinen Kopf möchte.

Avatar von

Zur Berechnung des Konvergenzradius gibt es zwei Formeln, beide findest du hier:

https://de.m.wikipedia.org/wiki/Konvergenzradius

1 Antwort

+2 Daumen
 
Beste Antwort

Hallo,

es gilt:$$\rho :=\limsup\limits_{k\to\infty}\sqrt[k]{\left |\frac{(-1)^k}{4k+1} \right |}=\limsup\limits_{k\to\infty}\frac{1}{\sqrt[k]{k(4+1/k)}}=\limsup\limits_{k\to\infty}\frac{1}{\sqrt[k]{k}\sqrt[k]{4+1/k}}=1$$ mit Konvergenzradius \(r:=\begin{cases}\frac{1}{\rho}, \text{ falls } \rho \in (0,\infty) \\ \infty, \text{ falls } \rho =0 \\ 0 , \text{ falls } \rho =\infty\end{cases}\),  also \(r=1\).

Für diese Rechnung solltest du gezeigt haben, dass \(\lim\limits_{k\to\infty}\sqrt[k]{k}=1\) und \(\lim\limits_{k\to\infty}\sqrt[k]{a}=1\) für \(a\in \mathbb{R}\), \(a>0\).

Du hast also nun ein Konvergenzintervall \(-1<x<1\). Für die Untersuchung an den Randpunkten, pickst du dir einen, z. B. \(x=1\), dann folgt:$$\sum \limits_{k=1}^{\infty} \frac{(-1)^{k}}{4 k+1} 1^{k}=\sum \limits_{k=1}^{\infty} \frac{(-1)^{k}}{4 k+1}=\sum \limits_{k=1}^{\infty} (-1)^{k}\frac{1}{4 k+1}$$ Dieser Ausdruck konvergiert, da \(\frac{1}{4 k+1}\) eine monoton fallende, relle Nullfolge ist. (Leibniz-Kriterium) Du darfst \(x=-1\) mal selbst probieren! :)

Avatar von 28 k

Da habe ich nun \( \sum\limits_{k=1}^{\infty}{\frac{1}{4k+1}} \) heraus. Wie weise ich denn nun nach, ob dies konvergent oder divergent ist?...

Vergleiche die Reihe mit der harmonischen Reihe.

Ich verstehs einfach nicht.

Trotzdem danke! :)

Dann schau mal nach "Minorantenkriterium" :)

Ein anderes Problem?

Stell deine Frage

Willkommen bei der Mathelounge! Stell deine Frage einfach und kostenlos

x
Made by a lovely community